If $Q inmathrmSpec(B)$ is the unique prime ideal lying over $P inmathrmSpec(A)$, then $B_P=B_Q$. The 2019 Stack Overflow Developer Survey Results Are In Announcing the arrival of Valued Associate #679: Cesar Manara Planned maintenance scheduled April 17/18, 2019 at 00:00UTC (8:00pm US/Eastern)intersections of powers of primes lying over a prime in a Galois extensionComputing the “lying over”, “going up”, “going down” ideals.If $mathopmathrmSpecA$ is not connected then there is a nontrivial idempotentExistence of prime ideal lying over $p implies pS cap R=p $?intersection of localization with the base fieldQuestion about correspondence of ideals in rings of fractions (if $S^-1q'$ is a maximal ideal containing $S^-1b,$ then must $q'$ contain $b$?)A question of the uniqueness of the prime ideal lying over in an integral extension (Corollary 5.9 in Atiyah's Commutative Algebra)$B subseteq A$ be an integral extension of integral domains where $B$ is normal; if $P in Spec A$ and $Q=Pcap B$, then $dim A_P=dim B_Q$?If $B$ is integral over $A$ and there is only one $P$ over $mathfrak p$, then $B_P=B_mathfrak p$?Checking whether certain lying-over prime is isolated

Can a flute soloist sit?

How to handle characters who are more educated than the author?

The following signatures were invalid: EXPKEYSIG 1397BC53640DB551

Are spiders unable to hurt humans, especially very small spiders?

Keeping a retro style to sci-fi spaceships?

One-dimensional Japanese puzzle

Am I ethically obligated to go into work on an off day if the reason is sudden?

What to do when moving next to a bird sanctuary with a loosely-domesticated cat?

How did the audience guess the pentatonic scale in Bobby McFerrin's presentation?

Is every episode of "Where are my Pants?" identical?

Could an empire control the whole planet with today's comunication methods?

Do warforged have souls?

Huge performance difference of the command find with and without using %M option to show permissions

Accepted by European university, rejected by all American ones I applied to? Possible reasons?

My body leaves; my core can stay

Can each chord in a progression create its own key?

Did the UK government pay "millions and millions of dollars" to try to snag Julian Assange?

What was the last x86 CPU that did not have the x87 floating-point unit built in?

US Healthcare consultation for visitors

Is there a way to generate uniformly distributed points on a sphere from a fixed amount of random real numbers per point?

First use of “packing” as in carrying a gun

Why doesn't a hydraulic lever violate conservation of energy?

how can a perfect fourth interval be considered either consonant or dissonant?

Did the new image of black hole confirm the general theory of relativity?



If $Q inmathrmSpec(B)$ is the unique prime ideal lying over $P inmathrmSpec(A)$, then $B_P=B_Q$.



The 2019 Stack Overflow Developer Survey Results Are In
Announcing the arrival of Valued Associate #679: Cesar Manara
Planned maintenance scheduled April 17/18, 2019 at 00:00UTC (8:00pm US/Eastern)intersections of powers of primes lying over a prime in a Galois extensionComputing the “lying over”, “going up”, “going down” ideals.If $mathopmathrmSpecA$ is not connected then there is a nontrivial idempotentExistence of prime ideal lying over $p implies pS cap R=p $?intersection of localization with the base fieldQuestion about correspondence of ideals in rings of fractions (if $S^-1q'$ is a maximal ideal containing $S^-1b,$ then must $q'$ contain $b$?)A question of the uniqueness of the prime ideal lying over in an integral extension (Corollary 5.9 in Atiyah's Commutative Algebra)$B subseteq A$ be an integral extension of integral domains where $B$ is normal; if $P in Spec A$ and $Q=Pcap B$, then $dim A_P=dim B_Q$?If $B$ is integral over $A$ and there is only one $P$ over $mathfrak p$, then $B_P=B_mathfrak p$?Checking whether certain lying-over prime is isolated










1












$begingroup$



Let $A subseteq B$ be an integral extension of integral domains. Suppose $P in mathrmSpec(A)$ and $Q in mathrmSpec(B)$ is the unique prime ideal lying above $P$. Prove that $B_Q=B_P$.




My attempt: since $P=Qcap A subseteq Q$, we have $B_Q subseteq B_P$. For the converse, suppose $b/s in B_P$. Then $s notin P$. Since $P=Q cap A$, we have $s notin Q cap A$. That is, $s notin Q$ or $s notin A$. If it's the former, then that's what we want. If it's the latter, how should I arrive at a contradiction?



I don't know how to use the condition that $B$ is integral over $A$. If $s$ satisfies the equation $s^n+a_n-1s^n-1+cdots+a_1s+a_0=0$ for some $a_n-1,ldots,a_0 in A$. What can that do for me?



Can anyone give me a hint? Thank you!










share|cite|improve this question











$endgroup$











  • $begingroup$
    Actually it is obvious that $B_Psubseteq B_Q$. The other containement has to be proved.
    $endgroup$
    – user26857
    Apr 8 at 13:50
















1












$begingroup$



Let $A subseteq B$ be an integral extension of integral domains. Suppose $P in mathrmSpec(A)$ and $Q in mathrmSpec(B)$ is the unique prime ideal lying above $P$. Prove that $B_Q=B_P$.




My attempt: since $P=Qcap A subseteq Q$, we have $B_Q subseteq B_P$. For the converse, suppose $b/s in B_P$. Then $s notin P$. Since $P=Q cap A$, we have $s notin Q cap A$. That is, $s notin Q$ or $s notin A$. If it's the former, then that's what we want. If it's the latter, how should I arrive at a contradiction?



I don't know how to use the condition that $B$ is integral over $A$. If $s$ satisfies the equation $s^n+a_n-1s^n-1+cdots+a_1s+a_0=0$ for some $a_n-1,ldots,a_0 in A$. What can that do for me?



Can anyone give me a hint? Thank you!










share|cite|improve this question











$endgroup$











  • $begingroup$
    Actually it is obvious that $B_Psubseteq B_Q$. The other containement has to be proved.
    $endgroup$
    – user26857
    Apr 8 at 13:50














1












1








1





$begingroup$



Let $A subseteq B$ be an integral extension of integral domains. Suppose $P in mathrmSpec(A)$ and $Q in mathrmSpec(B)$ is the unique prime ideal lying above $P$. Prove that $B_Q=B_P$.




My attempt: since $P=Qcap A subseteq Q$, we have $B_Q subseteq B_P$. For the converse, suppose $b/s in B_P$. Then $s notin P$. Since $P=Q cap A$, we have $s notin Q cap A$. That is, $s notin Q$ or $s notin A$. If it's the former, then that's what we want. If it's the latter, how should I arrive at a contradiction?



I don't know how to use the condition that $B$ is integral over $A$. If $s$ satisfies the equation $s^n+a_n-1s^n-1+cdots+a_1s+a_0=0$ for some $a_n-1,ldots,a_0 in A$. What can that do for me?



Can anyone give me a hint? Thank you!










share|cite|improve this question











$endgroup$





Let $A subseteq B$ be an integral extension of integral domains. Suppose $P in mathrmSpec(A)$ and $Q in mathrmSpec(B)$ is the unique prime ideal lying above $P$. Prove that $B_Q=B_P$.




My attempt: since $P=Qcap A subseteq Q$, we have $B_Q subseteq B_P$. For the converse, suppose $b/s in B_P$. Then $s notin P$. Since $P=Q cap A$, we have $s notin Q cap A$. That is, $s notin Q$ or $s notin A$. If it's the former, then that's what we want. If it's the latter, how should I arrive at a contradiction?



I don't know how to use the condition that $B$ is integral over $A$. If $s$ satisfies the equation $s^n+a_n-1s^n-1+cdots+a_1s+a_0=0$ for some $a_n-1,ldots,a_0 in A$. What can that do for me?



Can anyone give me a hint? Thank you!







commutative-algebra localization integral-extensions






share|cite|improve this question















share|cite|improve this question













share|cite|improve this question




share|cite|improve this question








edited Apr 8 at 8:50









user26857

39.5k124284




39.5k124284










asked Apr 8 at 8:13









bbwbbw

53739




53739











  • $begingroup$
    Actually it is obvious that $B_Psubseteq B_Q$. The other containement has to be proved.
    $endgroup$
    – user26857
    Apr 8 at 13:50

















  • $begingroup$
    Actually it is obvious that $B_Psubseteq B_Q$. The other containement has to be proved.
    $endgroup$
    – user26857
    Apr 8 at 13:50
















$begingroup$
Actually it is obvious that $B_Psubseteq B_Q$. The other containement has to be proved.
$endgroup$
– user26857
Apr 8 at 13:50





$begingroup$
Actually it is obvious that $B_Psubseteq B_Q$. The other containement has to be proved.
$endgroup$
– user26857
Apr 8 at 13:50











2 Answers
2






active

oldest

votes


















1












$begingroup$

Look at $B_P|_A_P$ which is an integral extension. The fact that only one prime lies over $PA_P$ implies $B_P$ is local. Also observe that $QB_P$ has to be the unique maximal ideal of $B_P$. Thus anything outside $Q$ is a unit in $B_P$ and hence $B_Q$ =$B_P$






share|cite|improve this answer









$endgroup$












  • $begingroup$
    Why does $B_P$ is local implies $B_Q=B_P$? I couldn't follow this step.
    $endgroup$
    – bbw
    Apr 8 at 9:09










  • $begingroup$
    @bbw $B_Psubset B_Q$ since you are inverting more elements. Now take any element $s$ with $s in B-Q$. So $s$ is outside $QB_P$ and hence a unit in $B_P$ Thus all elements of $B-Q$ are already inverted.
    $endgroup$
    – Soumik Ghosh
    Apr 8 at 9:20


















1












$begingroup$

Clearly $B_Psubseteq B_Q$. In order to prove that $B_P=B_Q$ notice that $B_Q$ is the localization of $B_P$ at $QB_P$. Now let's prove that $B_P$ is a local ring and its maximal ideal is $QB_P$. Any prime ideal of $B_P$ is of the form $Q'B_P$ with $Q'cap Asubseteq P$. Since $Q$ is the only prime ideal lying over $P$ we have $Q'cap Asubsetneq P$ provided $Q'ne Q$. By Going Up there is a prime ideal $Q''$ of $B$ (strictly) containing $Q'$ and lying over $P$. By hypothesis $Q''=Q$, and thus $Q'subsetneq Q$ proving that $QB_P$ is the only maximal ideal.






share|cite|improve this answer











$endgroup$












  • $begingroup$
    Why is $B_Q$ the localization of $B_P$ at $QB_P$? $B_Q$ is $(B setminus Q)^-1 B$ and the other is $(B_P setminus QB_P)^-1B_P$. And I still don't understand why $B_P$ is local $Rightarrow$ $B_P=B_Q$. Can you explain a little bit more?
    $endgroup$
    – bbw
    Apr 8 at 17:36











  • $begingroup$
    If $S=Asetminus P$, then $Qcap S=emptyset$ and a well known property of localization says that $(S^-1B)_S^-1Q$ is isomorphic to $B_Q$. But $S^-1B=B_P$, and $S^-1Q=QB_P$.
    $endgroup$
    – user26857
    Apr 8 at 20:07











  • $begingroup$
    If $R$ is a local ring with maximal ideal $mathfrak m$ then $R_mathfrak m=R$ since every element in $Rsetminusmathfrak m$ is invertible in $R$.
    $endgroup$
    – user26857
    Apr 8 at 20:09










  • $begingroup$
    Thank you for you explanation!
    $endgroup$
    – bbw
    Apr 8 at 23:38











Your Answer








StackExchange.ready(function()
var channelOptions =
tags: "".split(" "),
id: "69"
;
initTagRenderer("".split(" "), "".split(" "), channelOptions);

StackExchange.using("externalEditor", function()
// Have to fire editor after snippets, if snippets enabled
if (StackExchange.settings.snippets.snippetsEnabled)
StackExchange.using("snippets", function()
createEditor();
);

else
createEditor();

);

function createEditor()
StackExchange.prepareEditor(
heartbeatType: 'answer',
autoActivateHeartbeat: false,
convertImagesToLinks: true,
noModals: true,
showLowRepImageUploadWarning: true,
reputationToPostImages: 10,
bindNavPrevention: true,
postfix: "",
imageUploader:
brandingHtml: "Powered by u003ca class="icon-imgur-white" href="https://imgur.com/"u003eu003c/au003e",
contentPolicyHtml: "User contributions licensed under u003ca href="https://creativecommons.org/licenses/by-sa/3.0/"u003ecc by-sa 3.0 with attribution requiredu003c/au003e u003ca href="https://stackoverflow.com/legal/content-policy"u003e(content policy)u003c/au003e",
allowUrls: true
,
noCode: true, onDemand: true,
discardSelector: ".discard-answer"
,immediatelyShowMarkdownHelp:true
);



);













draft saved

draft discarded


















StackExchange.ready(
function ()
StackExchange.openid.initPostLogin('.new-post-login', 'https%3a%2f%2fmath.stackexchange.com%2fquestions%2f3179322%2fif-q-in-mathrmspecb-is-the-unique-prime-ideal-lying-over-p-in-mathrmsp%23new-answer', 'question_page');

);

Post as a guest















Required, but never shown

























2 Answers
2






active

oldest

votes








2 Answers
2






active

oldest

votes









active

oldest

votes






active

oldest

votes









1












$begingroup$

Look at $B_P|_A_P$ which is an integral extension. The fact that only one prime lies over $PA_P$ implies $B_P$ is local. Also observe that $QB_P$ has to be the unique maximal ideal of $B_P$. Thus anything outside $Q$ is a unit in $B_P$ and hence $B_Q$ =$B_P$






share|cite|improve this answer









$endgroup$












  • $begingroup$
    Why does $B_P$ is local implies $B_Q=B_P$? I couldn't follow this step.
    $endgroup$
    – bbw
    Apr 8 at 9:09










  • $begingroup$
    @bbw $B_Psubset B_Q$ since you are inverting more elements. Now take any element $s$ with $s in B-Q$. So $s$ is outside $QB_P$ and hence a unit in $B_P$ Thus all elements of $B-Q$ are already inverted.
    $endgroup$
    – Soumik Ghosh
    Apr 8 at 9:20















1












$begingroup$

Look at $B_P|_A_P$ which is an integral extension. The fact that only one prime lies over $PA_P$ implies $B_P$ is local. Also observe that $QB_P$ has to be the unique maximal ideal of $B_P$. Thus anything outside $Q$ is a unit in $B_P$ and hence $B_Q$ =$B_P$






share|cite|improve this answer









$endgroup$












  • $begingroup$
    Why does $B_P$ is local implies $B_Q=B_P$? I couldn't follow this step.
    $endgroup$
    – bbw
    Apr 8 at 9:09










  • $begingroup$
    @bbw $B_Psubset B_Q$ since you are inverting more elements. Now take any element $s$ with $s in B-Q$. So $s$ is outside $QB_P$ and hence a unit in $B_P$ Thus all elements of $B-Q$ are already inverted.
    $endgroup$
    – Soumik Ghosh
    Apr 8 at 9:20













1












1








1





$begingroup$

Look at $B_P|_A_P$ which is an integral extension. The fact that only one prime lies over $PA_P$ implies $B_P$ is local. Also observe that $QB_P$ has to be the unique maximal ideal of $B_P$. Thus anything outside $Q$ is a unit in $B_P$ and hence $B_Q$ =$B_P$






share|cite|improve this answer









$endgroup$



Look at $B_P|_A_P$ which is an integral extension. The fact that only one prime lies over $PA_P$ implies $B_P$ is local. Also observe that $QB_P$ has to be the unique maximal ideal of $B_P$. Thus anything outside $Q$ is a unit in $B_P$ and hence $B_Q$ =$B_P$







share|cite|improve this answer












share|cite|improve this answer



share|cite|improve this answer










answered Apr 8 at 8:45









Soumik GhoshSoumik Ghosh

1,298112




1,298112











  • $begingroup$
    Why does $B_P$ is local implies $B_Q=B_P$? I couldn't follow this step.
    $endgroup$
    – bbw
    Apr 8 at 9:09










  • $begingroup$
    @bbw $B_Psubset B_Q$ since you are inverting more elements. Now take any element $s$ with $s in B-Q$. So $s$ is outside $QB_P$ and hence a unit in $B_P$ Thus all elements of $B-Q$ are already inverted.
    $endgroup$
    – Soumik Ghosh
    Apr 8 at 9:20
















  • $begingroup$
    Why does $B_P$ is local implies $B_Q=B_P$? I couldn't follow this step.
    $endgroup$
    – bbw
    Apr 8 at 9:09










  • $begingroup$
    @bbw $B_Psubset B_Q$ since you are inverting more elements. Now take any element $s$ with $s in B-Q$. So $s$ is outside $QB_P$ and hence a unit in $B_P$ Thus all elements of $B-Q$ are already inverted.
    $endgroup$
    – Soumik Ghosh
    Apr 8 at 9:20















$begingroup$
Why does $B_P$ is local implies $B_Q=B_P$? I couldn't follow this step.
$endgroup$
– bbw
Apr 8 at 9:09




$begingroup$
Why does $B_P$ is local implies $B_Q=B_P$? I couldn't follow this step.
$endgroup$
– bbw
Apr 8 at 9:09












$begingroup$
@bbw $B_Psubset B_Q$ since you are inverting more elements. Now take any element $s$ with $s in B-Q$. So $s$ is outside $QB_P$ and hence a unit in $B_P$ Thus all elements of $B-Q$ are already inverted.
$endgroup$
– Soumik Ghosh
Apr 8 at 9:20




$begingroup$
@bbw $B_Psubset B_Q$ since you are inverting more elements. Now take any element $s$ with $s in B-Q$. So $s$ is outside $QB_P$ and hence a unit in $B_P$ Thus all elements of $B-Q$ are already inverted.
$endgroup$
– Soumik Ghosh
Apr 8 at 9:20











1












$begingroup$

Clearly $B_Psubseteq B_Q$. In order to prove that $B_P=B_Q$ notice that $B_Q$ is the localization of $B_P$ at $QB_P$. Now let's prove that $B_P$ is a local ring and its maximal ideal is $QB_P$. Any prime ideal of $B_P$ is of the form $Q'B_P$ with $Q'cap Asubseteq P$. Since $Q$ is the only prime ideal lying over $P$ we have $Q'cap Asubsetneq P$ provided $Q'ne Q$. By Going Up there is a prime ideal $Q''$ of $B$ (strictly) containing $Q'$ and lying over $P$. By hypothesis $Q''=Q$, and thus $Q'subsetneq Q$ proving that $QB_P$ is the only maximal ideal.






share|cite|improve this answer











$endgroup$












  • $begingroup$
    Why is $B_Q$ the localization of $B_P$ at $QB_P$? $B_Q$ is $(B setminus Q)^-1 B$ and the other is $(B_P setminus QB_P)^-1B_P$. And I still don't understand why $B_P$ is local $Rightarrow$ $B_P=B_Q$. Can you explain a little bit more?
    $endgroup$
    – bbw
    Apr 8 at 17:36











  • $begingroup$
    If $S=Asetminus P$, then $Qcap S=emptyset$ and a well known property of localization says that $(S^-1B)_S^-1Q$ is isomorphic to $B_Q$. But $S^-1B=B_P$, and $S^-1Q=QB_P$.
    $endgroup$
    – user26857
    Apr 8 at 20:07











  • $begingroup$
    If $R$ is a local ring with maximal ideal $mathfrak m$ then $R_mathfrak m=R$ since every element in $Rsetminusmathfrak m$ is invertible in $R$.
    $endgroup$
    – user26857
    Apr 8 at 20:09










  • $begingroup$
    Thank you for you explanation!
    $endgroup$
    – bbw
    Apr 8 at 23:38















1












$begingroup$

Clearly $B_Psubseteq B_Q$. In order to prove that $B_P=B_Q$ notice that $B_Q$ is the localization of $B_P$ at $QB_P$. Now let's prove that $B_P$ is a local ring and its maximal ideal is $QB_P$. Any prime ideal of $B_P$ is of the form $Q'B_P$ with $Q'cap Asubseteq P$. Since $Q$ is the only prime ideal lying over $P$ we have $Q'cap Asubsetneq P$ provided $Q'ne Q$. By Going Up there is a prime ideal $Q''$ of $B$ (strictly) containing $Q'$ and lying over $P$. By hypothesis $Q''=Q$, and thus $Q'subsetneq Q$ proving that $QB_P$ is the only maximal ideal.






share|cite|improve this answer











$endgroup$












  • $begingroup$
    Why is $B_Q$ the localization of $B_P$ at $QB_P$? $B_Q$ is $(B setminus Q)^-1 B$ and the other is $(B_P setminus QB_P)^-1B_P$. And I still don't understand why $B_P$ is local $Rightarrow$ $B_P=B_Q$. Can you explain a little bit more?
    $endgroup$
    – bbw
    Apr 8 at 17:36











  • $begingroup$
    If $S=Asetminus P$, then $Qcap S=emptyset$ and a well known property of localization says that $(S^-1B)_S^-1Q$ is isomorphic to $B_Q$. But $S^-1B=B_P$, and $S^-1Q=QB_P$.
    $endgroup$
    – user26857
    Apr 8 at 20:07











  • $begingroup$
    If $R$ is a local ring with maximal ideal $mathfrak m$ then $R_mathfrak m=R$ since every element in $Rsetminusmathfrak m$ is invertible in $R$.
    $endgroup$
    – user26857
    Apr 8 at 20:09










  • $begingroup$
    Thank you for you explanation!
    $endgroup$
    – bbw
    Apr 8 at 23:38













1












1








1





$begingroup$

Clearly $B_Psubseteq B_Q$. In order to prove that $B_P=B_Q$ notice that $B_Q$ is the localization of $B_P$ at $QB_P$. Now let's prove that $B_P$ is a local ring and its maximal ideal is $QB_P$. Any prime ideal of $B_P$ is of the form $Q'B_P$ with $Q'cap Asubseteq P$. Since $Q$ is the only prime ideal lying over $P$ we have $Q'cap Asubsetneq P$ provided $Q'ne Q$. By Going Up there is a prime ideal $Q''$ of $B$ (strictly) containing $Q'$ and lying over $P$. By hypothesis $Q''=Q$, and thus $Q'subsetneq Q$ proving that $QB_P$ is the only maximal ideal.






share|cite|improve this answer











$endgroup$



Clearly $B_Psubseteq B_Q$. In order to prove that $B_P=B_Q$ notice that $B_Q$ is the localization of $B_P$ at $QB_P$. Now let's prove that $B_P$ is a local ring and its maximal ideal is $QB_P$. Any prime ideal of $B_P$ is of the form $Q'B_P$ with $Q'cap Asubseteq P$. Since $Q$ is the only prime ideal lying over $P$ we have $Q'cap Asubsetneq P$ provided $Q'ne Q$. By Going Up there is a prime ideal $Q''$ of $B$ (strictly) containing $Q'$ and lying over $P$. By hypothesis $Q''=Q$, and thus $Q'subsetneq Q$ proving that $QB_P$ is the only maximal ideal.







share|cite|improve this answer














share|cite|improve this answer



share|cite|improve this answer








edited Apr 8 at 14:12

























answered Apr 8 at 14:06









user26857user26857

39.5k124284




39.5k124284











  • $begingroup$
    Why is $B_Q$ the localization of $B_P$ at $QB_P$? $B_Q$ is $(B setminus Q)^-1 B$ and the other is $(B_P setminus QB_P)^-1B_P$. And I still don't understand why $B_P$ is local $Rightarrow$ $B_P=B_Q$. Can you explain a little bit more?
    $endgroup$
    – bbw
    Apr 8 at 17:36











  • $begingroup$
    If $S=Asetminus P$, then $Qcap S=emptyset$ and a well known property of localization says that $(S^-1B)_S^-1Q$ is isomorphic to $B_Q$. But $S^-1B=B_P$, and $S^-1Q=QB_P$.
    $endgroup$
    – user26857
    Apr 8 at 20:07











  • $begingroup$
    If $R$ is a local ring with maximal ideal $mathfrak m$ then $R_mathfrak m=R$ since every element in $Rsetminusmathfrak m$ is invertible in $R$.
    $endgroup$
    – user26857
    Apr 8 at 20:09










  • $begingroup$
    Thank you for you explanation!
    $endgroup$
    – bbw
    Apr 8 at 23:38
















  • $begingroup$
    Why is $B_Q$ the localization of $B_P$ at $QB_P$? $B_Q$ is $(B setminus Q)^-1 B$ and the other is $(B_P setminus QB_P)^-1B_P$. And I still don't understand why $B_P$ is local $Rightarrow$ $B_P=B_Q$. Can you explain a little bit more?
    $endgroup$
    – bbw
    Apr 8 at 17:36











  • $begingroup$
    If $S=Asetminus P$, then $Qcap S=emptyset$ and a well known property of localization says that $(S^-1B)_S^-1Q$ is isomorphic to $B_Q$. But $S^-1B=B_P$, and $S^-1Q=QB_P$.
    $endgroup$
    – user26857
    Apr 8 at 20:07











  • $begingroup$
    If $R$ is a local ring with maximal ideal $mathfrak m$ then $R_mathfrak m=R$ since every element in $Rsetminusmathfrak m$ is invertible in $R$.
    $endgroup$
    – user26857
    Apr 8 at 20:09










  • $begingroup$
    Thank you for you explanation!
    $endgroup$
    – bbw
    Apr 8 at 23:38















$begingroup$
Why is $B_Q$ the localization of $B_P$ at $QB_P$? $B_Q$ is $(B setminus Q)^-1 B$ and the other is $(B_P setminus QB_P)^-1B_P$. And I still don't understand why $B_P$ is local $Rightarrow$ $B_P=B_Q$. Can you explain a little bit more?
$endgroup$
– bbw
Apr 8 at 17:36





$begingroup$
Why is $B_Q$ the localization of $B_P$ at $QB_P$? $B_Q$ is $(B setminus Q)^-1 B$ and the other is $(B_P setminus QB_P)^-1B_P$. And I still don't understand why $B_P$ is local $Rightarrow$ $B_P=B_Q$. Can you explain a little bit more?
$endgroup$
– bbw
Apr 8 at 17:36













$begingroup$
If $S=Asetminus P$, then $Qcap S=emptyset$ and a well known property of localization says that $(S^-1B)_S^-1Q$ is isomorphic to $B_Q$. But $S^-1B=B_P$, and $S^-1Q=QB_P$.
$endgroup$
– user26857
Apr 8 at 20:07





$begingroup$
If $S=Asetminus P$, then $Qcap S=emptyset$ and a well known property of localization says that $(S^-1B)_S^-1Q$ is isomorphic to $B_Q$. But $S^-1B=B_P$, and $S^-1Q=QB_P$.
$endgroup$
– user26857
Apr 8 at 20:07













$begingroup$
If $R$ is a local ring with maximal ideal $mathfrak m$ then $R_mathfrak m=R$ since every element in $Rsetminusmathfrak m$ is invertible in $R$.
$endgroup$
– user26857
Apr 8 at 20:09




$begingroup$
If $R$ is a local ring with maximal ideal $mathfrak m$ then $R_mathfrak m=R$ since every element in $Rsetminusmathfrak m$ is invertible in $R$.
$endgroup$
– user26857
Apr 8 at 20:09












$begingroup$
Thank you for you explanation!
$endgroup$
– bbw
Apr 8 at 23:38




$begingroup$
Thank you for you explanation!
$endgroup$
– bbw
Apr 8 at 23:38

















draft saved

draft discarded
















































Thanks for contributing an answer to Mathematics Stack Exchange!


  • Please be sure to answer the question. Provide details and share your research!

But avoid


  • Asking for help, clarification, or responding to other answers.

  • Making statements based on opinion; back them up with references or personal experience.

Use MathJax to format equations. MathJax reference.


To learn more, see our tips on writing great answers.




draft saved


draft discarded














StackExchange.ready(
function ()
StackExchange.openid.initPostLogin('.new-post-login', 'https%3a%2f%2fmath.stackexchange.com%2fquestions%2f3179322%2fif-q-in-mathrmspecb-is-the-unique-prime-ideal-lying-over-p-in-mathrmsp%23new-answer', 'question_page');

);

Post as a guest















Required, but never shown





















































Required, but never shown














Required, but never shown












Required, but never shown







Required, but never shown

































Required, but never shown














Required, but never shown












Required, but never shown







Required, but never shown







Popular posts from this blog

Hidroelektrana Sadržaj Povijest | Podjela hidroelektrana | Snaga dobivena u hidroelektranama | Dijelovi hidroelektrane | Uloga hidroelektrana u suvremenom svijetu | Prednosti hidroelektrana | Nedostaci hidroelektrana | Države s najvećom proizvodnjom hidro-električne energije | Deset najvećih hidroelektrana u svijetu | Hidroelektrane u Hrvatskoj | Izvori | Poveznice | Vanjske poveznice | Navigacijski izbornikTechnical Report, Version 2Zajedničkom poslužiteljuHidroelektranaHEP Proizvodnja d.o.o. - Hidroelektrane u Hrvatskoj

WordPress Information needed

Oconto (Nebraska) Índice Demografia | Geografia | Localidades na vizinhança | Referências Ligações externas | Menu de navegação41° 8' 29" N 99° 45' 41" O41° 8' 29" N 99° 45' 41" OU.S. Census Bureau. Census 2000 Summary File 1U.S. Census Bureau. Estimativa da população (julho de 2006)U.S. Board on Geographic Names. Topical Gazetteers Populated Places. Gráficos do banco de dados de altitudes dos Estados Unidos da AméricaEstatísticas, mapas e outras informações sobre Oconto em city-data.com